Difference between revisions of "Mock AIME 4 2006-2007 Problems/Problem 14"

 
m
Line 5: Line 5:
 
{{solution}}
 
{{solution}}
  
 +
 +
 +
----
 +
 +
*[[Mock AIME 4 2006-2007 Problems/Problem 15| Next Problem]]
 +
*[[Mock AIME 4 2006-2007 Problems/Problem 13| Previous Problem]]
 
*[[Mock AIME 4 2006-2007 Problems]]
 
*[[Mock AIME 4 2006-2007 Problems]]

Revision as of 11:49, 16 January 2007

Problem

Let $x$ be the arithmetic mean of all positive integers $k<577$ such that

$k^4\equiv 144\pmod {577}$.

Find the greatest integer less than or equal to $x$.

Solution

This problem needs a solution. If you have a solution for it, please help us out by adding it.